exam 2 bcps auto

Réussis tes devoirs et examens dès maintenant avec Quizwiz!

Regarding fluid replacement, which of the following match is correct? A. 1 Liter of Dextrose solution provides 100 mls of intravascular volume replacement B. 1 Liter of normal saline provides 500 mls of intravascular volume replacement C. 1 Liter of 5% albumin will provide 500 mls of intravascular volume replacement D. 1 Liter of 25% albumin will provide 500 mls of intravascular volume replacement

Answer - A. 1 Liter of Dextrose solution provides 100 mls of intravascular volume replacement - 1 liter of dextrose will provide a 10% return on intravascular volume replacement. 1 liter of normal saline will be anticipated to increase volume by 250 mls, 5% albumin will provide a 1:1 return (each ml will increase intravascular volume by 1 ml). Albumin 25% would be anticipated to increase intravascular volume by 5X per 1 ml infused. (i.e. 1 ml infused, 5 mls increased intravascular volume).

In which of the following circumstances would ethinyl estradiol and norgestimate notbe contraindicated? A. A 40 year old female with a BMI of 33 B. A 38 year old who is a smokerC. A 32 year old female with uncontrolled hypertension D. A 41 year old with migraines/aura

Answer - A. A 40 year old female with a BMI of 33 - all of the above are contraindications with the exception of the patient with the elevated BMI. Using oral contraceptives in that situation would be acceptable.

Digoxin was initiated in a patient to help in management of atrial fibrillation. Which ofthe following would be least important to assess? A. AST B. Potassium C. Serum creatinine D. Pulse

Answer - A. AST - Pulse, kidney function, and potassium would all be important monitoring parameters. Since digoxin is primarily cleared by the kidney and wouldn't normally cause liver toxicity, this would be the least important. Pulse would obviously be important as it is a potential sign of toxicity and potassium is important due to the fact that toxicity is more likely in a hypokalemic state.

Which entity works under the FDA to review information from randomized controlled trials in a New Drug Application? A. CDER B. HHS C. NCQA D. NQF

Answer - A. CDER - (Center for Drug Evaluation and Research) is the group tasked with reviewing data from the New Drug Application. That group includes physicians, pharmacologists, statisticians, etc.

Which of the following regarding CMS and quality of care isfalse? A. CMS has adopted socioeconomic adjustments for all quality measure thattake into account patient population factors B. CMS payments are increasingly tied to quality of care C. Quality of care measurements include not just process models, but also actual outcomes of care D. Hospital quality measures are most frequently updated on a quarterly to annual basis

Answer - A. CMS has adopted socioeconomic adjustments for all quality measure that take into account patient population factors - At this time, CMS does not adjust the quality measure for different areas/patient populations/socioeconomic factors. They likely have not done this yet because they do not want to give providers excuses as to why they don't meet quality metrics.

A 33 year old female presents with breast pain in association with breast feeding. Sheis diagnosed with acute bacterial mastitis. This is her first ever episode. Which of the following would be the best choice for initialtherapy? A. Cephalexin B. Topical mupirocin C. Ciprofloxacin D. Metronidazole

Answer - A. Cephalexin - Beta-lactam oral therapy is the drug of choice for mastitis. If MRSA is suspected, clindamycin or sulfa/tmp may be utilized.

In a small study of 100 patients, they were given a new medication used to help with hypoglycemia. The interquartile range increase of the blood sugar was between 23-45. How much of the data is contained in thisrange? A. 25% B. 50% C. 75% D. Unable to determine

Answer - B. 50% - The interquartile range is the difference between the "quarters" or the difference between the 75% mark and the 25% mark. The answer would be 50% of the data is contained within this range.

LS has not been taking her simvastatin which is indicated given her risk profile. She stopped taking it because she was also doing a daily grapefruit and was concerned witha potential drug interaction. Which of the following would be the most appropriate recommendation to avoid this potential interaction? A. Change simvastatin to pravastatin B. Change simvastatin to lovastatin C. Change simvastatin to atorvastatin D. Change simvastatin to ezetimibe

Answer - A. Change simvastatin to pravastatin - Changing to pravastatin would be the most appropriate recommendation. Rosuvastatin would also be an acceptable alternative. With statin therapy being indicated, switching to ezetimibe would not be the most appropriate recommendation.

OS is a 51 year old male receiving enteral nutrition via gastrostomy tube due to a motor vehicle accident. His current medications include warfarin, cholestyramine, levothyroxine, and lisinopril. Current diagnoses include; atrial fibrillation, hypertension, hypothyroidism, and diarrhea. Which medication should not be given via gastrostomy tube? A. Cholestyramine B. Warfarin C. Levothyroxine D. Lisinopril

Answer - A. Cholestyramine - For fear of clogging the tube, cholestyramine should never be given via g-tube. The other medications should be able to be crushed and given as appropriate. KM is a 31 year old male with an extensive addiction history. He has polysubstance abuse including opioids, alcoholism and methamphetamine. His current medications include: • Thiamine 100 mg daily • Folic acid 1 mg daily • MS Contin 60 mg BID • Omeprazole 20 mg daily • Ibuprofen 400 mg TID prn • Diphenhydramine 50 mg at bedtime

A 34 year old female is diagnosed with bacterial vaginosis. Her past medical history includes GERD, opioid abuse, mild hepatic impairment, alcoholism, and HIV. Which ofthe following would be the most appropriate medication to recommend? A. Clindamycin B. Levofloxacin C. Metronidazole D. Cephalexin

Answer - A. Clindamycin - Clindamycin and metronidazole are the usual drugs of choice here. With the patient's alcoholism, avoiding metronidazole would be the safest bet and using clindamycin would be more appropriate.

Which of the following would be considered the highest strength ofevidence? A. Cohort B. Case studies C. Expert opinionD. Observationalsurveys

Answer - A. Cohort - The Cohort study would have the highest strength of clinical evidence of the above listed. Expert opinion is considered the lowest strength.

Why can't rifampin be used as monotherapy for infections requiring longer courses of therapy? A. Development ofresistance B. Risk of drug interactions C. Elevations in LFT's are likely D. Rifampin is often used as booster with other antibiotics because it can raise concentrations of numerous antibiotics

Answer - A. Development of resistance - Rifampin should not be used as monotherapy because resistance can develop. There are numerous drug interactions, but most often rifampin will reduce concentrations of other medications because it is an enzyme inducer.

Patients with which deficiency may be at higher risk of toxicity from5-fluourouracil? A. Dihydropyrimidine dehydrogenase B. Sodium-potassium ATPase C. P-Glycoprotein D. Alcohol dehydrogenase

Answer - A. Dihydropyrimidine dehydrogenase - Patients with dihydropyrimidine dehydrogenase deficiency are at higher risk of toxicity from 5-FU and Capecitabine.

JS is a 45 year old male who has been anticoagulated with heparin for the last five days. He is currently sedated and in intensive care. The hospitalist is suspecting that the patient is experiencing heparin induced thrombocytopenia given his most recent labs. She believes that anticoagulation is necessary. What is the appropriate course of action? A. Discontinue heparin and initiate argatroban B. Continue current heparin with a dose reduction of 50% C. Discontinue heparin and initiate warfarin D. Discontinue heparin and initiate enoxaparin

Answer - A. Discontinue heparin and initiate argatroban - With heparin induced thrombocytopenia, all heparin products should be discontinued including LMWH (enoxaparin). Warfarin will take some time to take effect and is given orally as well, not an appropriate switch at this time. The best answer would be to switch to argatroban.

A 65 year old female has rheumatoid arthritis, early onset Alzheimer's, iron deficiency, GERD, PUD, and PVD. She is reporting an increase in urinary incontinence. Which medication would be the most likely culprit? A. Donepezil B. Memantine C. Cilostazol D. Lansoprazole

Answer - A. Donepezil - Acetylcholinesterase inhibitors can contribute to urinary incontinence (much like the opposing anticholinergics are used to help urinary incontinence). Memantine, cilostazol, and lansoprazole would likely not affect this.

Which of the following would not be a factor in considering if a patient should be hospitalized for pneumonia? A. Elevated Bilirubin B. Uremia C. Age D. Respiratory rate

Answer - A. Elevated Bilirubin - In the curb 65 criteria, bilirubin is not a determining factor in the risk calculation of whether a patient should be hospitalized. Confusion, uremia, respiratory rate, blood pressure, and age are all considerations.

Which of the following physiologic changes would be least likely in a geriatricpatient? A. GI hypermotility B. Skin thinning C. Reduced albumin D. Increased body fat

Answer - A. GI Hypermotility - GI motility tends to reduce as we age and can be exacerbated by diseases like diabetic gastroparesis as well as have medication related causes like anticholinergics etc.

NE is a 63 year old male who presents to the emergency department with severe upper GI bleeding. He is very lethargic and can hardly walk even with assistance. BP = 75/38, hemoglobin = 6.3, hematocrit = 19, platelets = 145K. Upon assessment, the attending physician is suspecting esophageal ulceration. Which of the following interventions would be least appropriate? A. Give PO Pantoprazole 40 mg stat B. Utilize normal saline boluses as needed to help withhypotension C. Recommend initiating transfusion D. Hold any antiplatelet/anticoagulantmedications

Answer - A. Give PO Pantoprazole 40 mg stat - In these severe of a case of likely upper GI bleeding, we would not want to be giving PO medications. Saline, transfusion, and stopping medications that can increase the risk of bleeding would all be appropriate considerations in the management of this scenario. The PPI should be given via IV, not orally.

The attending physician is utilizing atracurium for neuromuscular blockade. The physician is not obtaining the desired blockade that she wants. Which of the following medications could potentially prevent this from happening? A. Amiodarone B. Phenytoin C. Lithium D. Furosemide

Answer - B. Phenytoin - Enzyme inducers phenytoin and carbamazepine can potentially reduce the effectiveness of non-depolarizing agents like Atracurium.

In the management of acute ischemic stroke which of the following would be part ofthe exclusion criteria for use of alteplase? A. Glucose level = 48 B. Platelet Count 145,000 C. Blood pressure 162/82 D. Urinary tract infection and sulfamethoxazole/trimethoprim treatment about2 weeks ago

Answer - A. Glucose level = 48 - The low glucose level would be the exclusion criteria here as hypoglycemia could potentially cause symptoms similar to stroke. Getting this blood sugar up first would be the primary clinical goal to ensure this isn't causing the symptoms. A concerning platelet level is typically considered 100k or less. Blood pressure cut off is considered 185/110 and the urinary tract infection would not play a role here.

A 38 year old patient with hypertension, GERD, and Type 1 diabetes was camping and found to be unresponsive. Upon assessment, he was found to be severely hypothermic. In the rewarming of this patient, which of the following would most likelyoccur?A. Hypoglycemia B. Alkalosis C. Hypertensive crisis D. Thyroid storm

Answer - A. Hypoglycemia - As rewarming occurs in a patient with type 1 diabetes, the most challenging problem that would likely occur would be hypoglycemia. Vasodilation can also happen leading to hypotension, generally not a hypertensive crisis.

Which of the following adverse effects would not be anticipated withempagliflozin? A. Increase in blood pressure B. Increased risk of urinary tractinfections C. Increased urine output D. Increase risk of bone fractures

Answer - A. Increase in blood pressure - The way the SGLT2 inhibitors work, they actually have a mild diuretic type effect. Increased urine output would be anticipated. There are also warnings about potential for urinary tract infections and increased risk of bone fractures. Hypotension is more likely with empagliflozin than hypertension.

Which of the following would not be a risk factor fordrug resistant community acquired pneumonia? A. Knee surgery requiring beta-lactam prophylaxis about 9 monthsago B. Exposure to a child at a daycare centerC. Age>65 D. Residence at a long term care facility

Answer - A. Knee surgery requiring beta-lactam prophylaxis about 9 months ago - Typically, recent antibiotic use is considered within the 3-6 month range. Exposure at the knee surgery and without any other antibiotic exposure in the 9 month window would not make this a potential risk factor. Exposure to children at a daycare center, age, and residence at a long term care facility would all be potential risk factors for drug resistant bacteria.

There are numerous drugs that are highly protein bound and can be substantially impacted by changes in serum albumin. Which of the following would be least likely to have clinically significant changes with a drop in albumin? A. Levetiracetam B. Warfarin C. Phenytoin D. Valproic acid

Answer - A. Levetiracetam - is not highly protein bound and would be least likely to have clinically significant changes based upon a change in serum albumin level. Phenytoin, warfarin, and valproic acid are classic examples of medications that changes in serum albumin can greatly impact free fraction and potentially lead to toxicity.

KL has recently been diagnosed as HIV+ and is set to begin antiretroviral therapy. She has a past medical history of Type 2 diabetes, hyperlipidemia, and obesity. Which of the following agents would place her at highest risk of exacerbatinghyperglycemia? A. Lopinavir/ritonavir B. Efavirenz C. Tenofovir D. Emtricitabine

Answer - A. Lopinavir/ritonavir - The protease inhibitors are going to be most likely to have a direct effect on increasing the risk of hyperglycemia. You may see an indirect effect from the NRTI's, but the most correct answer would be the PI's.

A 78 year old presents to the ED with status epilepticus. His current chronic medications include: Levetiracetam 500 mg BID, Aspirin 81 mg daily, Lorazepam 0.5 mg BID prn, Atenolol 100 mg daily, and Hydrochlorothiazide 25 mg daily. Which of the following would be the most appropriate agent of choice? A. Lorazepam B. Fosphenytoin C. Valproic acid D. Phenobarbital

Answer - A. Lorazepam - Lorazepam/benzodiazepines are the first line agents for status epilepticus. Phenytoin is typically a second line agent. The prn lorazepam that the patient has on their medication list would not be a contraindication to using lorazepam in a status epilepticus situation.

Which medication would be an appropriate selection in ulcerative colitis, but not Crohn's disease? A. Mesalamine enema B. Sulfasalazine C. Azathioprine D. Infliximab

Answer - A. Mesalamine enema - would be an acceptable choice in ulcerative colitis which is typically just limited to the area of the colon. Crohn's disease can be found all throughout the GI tract making the enema formulation of minimal benefit.

Which of the following would not be consistent with Propofol infusionsyndrome? A. Metabolic alkalosis B. Risk is associated with higher dosages C. Bradycardia D. Rhabdomyolysis

Answer - A. Metabolic alkalosis - Rhabdomyolysis, bradycardia, metabolic acidosis, elevated lipids, and possible renal failure are consistent with Propofol infusion syndrome. Higher dosing for longer periods of time are associated with higher risk.

Exclusion criteria for a clinical trial would be best found in what section of the clinical study report? A. Methods B. Discussion C. Results D. Study objectives

Answer - A. Methods - The methods section contains how you did your study. This would include what criteria was used to include people in the study and what was used to exclude people from the study.

Investigators are analyzing the pharmacokinetics of levodopa in patients who are alcoholics versus patients that abstain from using alcohol. What type of variable would these different groups be considered? A. Nominal B. Ordinal C. Continuous D. Ratio

Answer - A. Nominal - In this scenario, separating alcoholics from non-alcoholics would be a nominal variable. Classic examples of nominal data involve the ability to "name" the patient to a specific group

Which of the following types of incontinence is BPH the most commoncause? A. Overflow B. Functional C. Stress D. Urge

Answer - A. Overflow - BPH commonly causes overflow incontinence by cause a blockage or obstruction for urine to exit the ureter. Anticholinergics, alpha agonists are a couple of common medications that could worsen these symptoms and this type of incontinence. Alpha blockers and 5 alpha reductase inhibitors are used to help manage BPH.

Naproxen would likely cause what type of kidney failure? A. Post-renal B. Pre-renal C. Acute tubular necrosis D. Immune hypersensitivity reaction

Answer - B. Pre-renal - NSAIDs, ACE inhibitors, etc. are likely to impact the flow to the kidney and induce renal failure by pre-renal mechanisms.

Which of the following is not a risk factor for development ofosteoporosis? A. Premenopausal B. Cigarette smoking C. Corticosteroid use D. Rheumatoid arthritis

Answer - A. Premenopausal - all of the above with the exception of premenopausal are risk factors for development of osteoporosis. Postmenopausal certainly is a risk factor, not premenopausal. A 59 year old male has a past medical history of systolic CHF. His current blood pressure is 154/92 and pulse is 69. He reports no edema at this time and lungs are clear with no breathing difficulties. Current medications include aspirin 81 mg daily, atorvastatin 20 mg daily, and Lisinopril 40 mg daily. Current Labs: GFR = 84 mls/min, K+ = 3.8, Sodium = 141, Calcium = 9.4.

A company has released its results of its most recent clinical trial. It was a double blind, placebo controlled trial involving the use of a new antihypertensive medication. The article describing this study and its results would be classified as...? A. Primary literature B. Secondary literature C. Tertiary literature D. None of the above

Answer - A. Primary literature - Individual clinical trials would be classified as primary literature. Secondary literature generally involves a compilation of numerous clinical trials while tertiary literature tends to be information material like textbooks.

Aripiprazole has been started in a patient with schizophrenia. This has been substantially beneficial, but the patient is experiencing troublesome akathisia. Which of the following medications would most likely help manage this side effect? A. Propranolol B. Gabapentin C. Fluoxetine D. Bromocriptine

Answer - A. Propranolol - Lipophilic beta blockers like propranolol and anticholinergics like benztropine are commonly used to help patients who experience akathisia, but benefit of the antipsychotic trumps the potential adverse effects.

A 63 year old female has a past history of CKD with hyperphosphatemia. Current labs: Phosphorus - 6.1 mg/dL, calcium 10.9 mg/dL, Potassium = 5.2 mEq/L, Sodium = 136 mEq/L. Which medication would be most appropriate to initiate? A. Sevelamer B. Calcium acetate C. Aluminum hydroxide D. Calcitriol

Answer - A. Sevelamer - would be the most appropriate addition for its phosphate binding activity. Calcium acetate is also a commonly used phosphate binder, but with an already elevated calcium, this wouldn't be appropriate. Aluminum based phosphate binders are avoided due to potential toxicity. Calcitriol is active vitamin D and could increase both calcium and phosphorus.

A 17 year old female presents for a wellness checkup prior to going to college. Shehas never received the HPV vaccine. Which of the following is the most appropriate recommendation? A. She should receive 3 doses of HPV vaccine at 0, 2, and 6months B. She should receive a 2 dose course of HPV vaccine at 0 and 6months C. She should receive a 2 dose course of HPV vaccine at 0 and 6 months only if she reports sexual activity D. She should receive 3 doses of HPV vaccine at 0, 2, and 6 months only ifshe reports sexual activity

Answer - A. She should receive 3 doses of HPV vaccine at 0, 2, and 6 months - Guidelines allow for patients under 15 to receive a two dose course. Patients over 15 should receive the usual three dose course. Reported sexual activity should not be a factor in getting children vaccinated.

He has recently started naltrexone for his alcoholism. Which of the following effects would likely be anticipated from starting thismedication? A. Sweating B. Constipation C. Sedation D. Edema

Answer - A. Sweating - Naltrexone is utilized for management of alcoholism, but would be a risky addition in this patient for alcohol abuse as you are likely to put this patient into opioid withdrawal being on a decent sized dose of morphine. Naltrexone is a mu antagonist. Sweating would be the most likely symptom of opioid withdrawal. Anxiety, diarrhea, insomnia, etc. would be other potential symptoms of opioid withdrawal.

A 54 year old male presents to the ED with chest pain. It has been happening on and off for the last couple of days and seems to have gotten a little worse today. EKG reveals normal findings. Labs: creatinine 1.1, potassium = 4.8, sodium = 141, pH = 7.38, chloride = 101, Troponin and CK-MB are reported to be within normal limits. He is diagnosed with unstable angina. Which of the following assessments would be most appropriate to determine if early invasive strategies would be appropriate? A. TIMI B. CHADS2Vasc C. Anion Gap D. ASCVD risk score

Answer - A. TIMI - scoring would be most appropriate to assess if early invasive strategies would be appropriate. ASCVD risk scoring is utilized to help determine if statin and aspirin therapy is appropriate in primary prevention. CHADS2Vasc has to do with evaluating stroke risk in atrial fibrillation, and anion gap would be of no benefit in this scenario as the patient has a normal pH.

When presenting information to a group of patients, which of the following would bethe least important thing to consider? A. Time of day for the presentation B. Potential language barriers C. Education level/healthcare background of the patients being educated D. Size of the group

Answer - A. Time of day for the presentation - The time of the day for the presentation is probably not going to have an important impact on the information presented. The size of the group, potential language barriers, and the education level of the patients you are presenting the information to would be significant things to consider and may determine how you tailor your educational approach.

A 36 year old male has been to the emergency department 3 times in the last 6 months for migraine headaches. Prophylaxis is indicated. The patient stated he has been on a chronic medication in the past for his headaches. He reports that he felt he was in a brain fog and that it made him feel "stupid". Which medication would be most likely be referring to? A. Topiramate B. Lamotrigine C. Venlafaxine D. Metoprolol

Answer - A. Topiramate - is often used for migraines and can cause significant cognitive slowing or in this case, make the patient feel "stupid". This is the most likely medication that would cause this. Metoprolol tends to cause more fatigue than cognitive changes.

200 patients were given a new vaccine for influenza and 200 patients were given placebo. 20 patients in the vaccine group got influenza. 45 patients in the placebo group got influenza. What is the relative risk reduction? A. 0.125 B. 0.44 C. 2.25 D. 1.25

Answer - B. 0.44 - Relative risk is the (%treatment group with influenza/% placebo group with influenza) = 10%/22.5 or 0.1/0.225 = 0.44. If relative risk is less than one, it indicates a protective effect from the treatment. If it is greater than one, it indicates that treatment increases the risk of the outcome.

First order kinetics is best described by: A. 50 mg of a drug is removed by the body every hour regardless ofthe concentration B. 50% of a drug is removed by the body every hour C. The rate of elimination exponentially decreases with increasing concentration D. None of the above

Answer - B. 50% of a drug is removed by the body every hour - In first order kinetics, a constant percentage of the drug is removed over time. This is the most common type of pharmacokinetic model and also holds true to the idea that in 5 halflives, the drug is considered virtually eliminated from the body.

Which entity is responsible for monitoring and maintaining the National Guideline Clearinghouse? A. The Joint Commission B. AHRQ C. FDA D. DEA

Answer - B. AHRQ - The Agency for Healthcare Research and Quality provides the National Guideline Clearinghouse. It is a database that provides evidence based resources that can be for public use.

A 55 year old male has a past medical history of fibromyalgia, GERD, idiopathic thrombocytopenia, CKD stage 3, and constipation. He is experiencing joint stiffness in both his knees and hands. Non-drug therapy like heat and ice has been unsuccessful. No inflammation is reported and the primary care provider feels this is generalized osteoarthritis. Which initial recommendation would be most appropriate? A. Ibuprofen 200-400 mg every 4 hours as neededB. Acetaminophen 500-1,000 mg every 6 hours as needed C. Tramadol 50 mg four times per day as needed D. Oxycodone 5 mg every four hours as needed

Answer - B. Acetaminophen 500-1,000 mg every 6 hours as needed - This appears to be an initial diagnosis of osteoarthritis. Acetaminophen would be the best choice in the patient. NSAIDs could negatively impact kidney disease and opioids wouldn't be first line therapy.

1. WA is a 34 year old male who has been seen in the ED twice in the last two months for asthma exacerbations. His asthma gets worse during spring and fall seasons and upon questioning, that is what he attributes the recent ED visits to. He is currently taking albuterol on average 2-4 times per day and is on budesonide/formoterol160mcg/4.5mcg two inhalations twice daily. Which of the following would be the next beststep?A. Increase budesonide/formoterol dose B. Add montelukast C. Change albuterol to albuterol/ipratropium combination D. Discontinue budesonide/formoterol and start fluticasone/salmeterol 250/50 twice daily

Answer - B. Add montelukast - Given the seasonal nature of this patient's trouble, it would make sense to try to add montelukast and see if that helps with allergies as well as the asthma. Budesonide/formoterol is already at max dose so increasing wouldn't be appropriate. Changing the albuterol and adding ipratropium likely wouldn't do anything for improved control. Switching to fluticasone/salmeterol likely would not improve the outcome.

Regarding members of an IRB committee, which of the following is leastaccurate? A. IRB members should not be paid by the institution to avoid potential conflictsof interest B. An IRB can be a small as 3 members as long as there is a documented rationale C. A mix of men and women shall be recommended and notrequired D. At least one member should work in scientific areas and one member should have a focus on non-scientific areas

Answer - B. An IRB can be a small as 3 members as long as there is a documented rationale - An IRB should be at least five members. An effort should be made to make sure both genders are represented. Paying IRB members is allowed including reimbursements for mileage etc. A variety of backgrounds is ideal for an IRB, but it is recommended to have at least one member from scientific work background and one member not be from a non-scientific work background

A 46 year old male who is HIV+ has had an increased difficulty with shortness of breath. He is diagnosed with Legionnaire's disease. Which of the following would be the most appropriate recommendation fortreatment? A. Sulfamethoxazole/trimethoprim B. Azithromycin C. Ceftriaxone D. Doxycycline

Answer - B. Azithromycin - (macrolide) is the usual drug of choice for Legionnaire's disease. Levofloxacin is a commonly used alternative.

Which of the following vitamin deficiencies would be most appropriate to assess for in a patient with neuropathy and anemia? A. Vitamin D B. B12 C. Vitamin K D. Vitamin C

Answer - B. B12 - deficiency can contribute to neuropathy and anemia. In addition, severe deficiency can lead to psychiatric complications like confusion.

A 35 year old male has been having pseudoparkinson's type symptoms from taking ziprasidone. The provider and patient feel that the antipsychotic continues to be warranted despite the potential adverse effects. Which of the following medications would be most appropriate to add? A. Levodopa/carbidopa B. Benztropine C. Selegiline D. Valproic acid

Answer - B. Benztropine - Anticholinergics like benztropine or trihexyphenidyl are added to reduce the incidence of EPS type side effects when antipsychotics are necessary. Medications for Parkinson's are typically not utilized and valproic acid would not have a role in reducing pseudoparkinsonism.

Ceftriaxone is typically avoided in newborns due to which of the following reasons? A. Hepatotoxicity B. Biliary sludge effect C. Cerebral edema risk D. Elevations in ammonia levels

Answer - B. Biliary sludge effect - Risk of biliary sludge is the reason why ceftriaxone is avoided in newborns.

In management of peripheral arterial disease, which of the following would be most beneficial in symptomatic physical limitations? A. Anticoagulation with warfarin B. Cilostazol C. Pentoxifylline D. Ibuprofen

Answer - B. Cilostazol has the best evidence at reducing symptoms of peripheral arterial disease. It tends to have more drug interactions than Pentoxifylline through CYP3A4 and also has warnings in heart failure. NSAIDs and anticoagulation are not indicated in the absence of other factors necessitating their use.

Which of the following medications would not be classified on the NIOSH list thathas special requirements for handling? A. Finasteride B. Clonidine C. Testosterone D. Leuprolide

Answer - B. Clonidine - The National Institute for Occupational Safety and Health sets up a list of medications that may cause harm to healthcare professionals if exposed to the medication. Finasteride, testosterone, leuprolide would all have potential negative implications on healthcare workers (usually highest risk to women of childbearing age) and should be handled/administered with appropriate safety precautions.

Which of the following risks is not increased with use of estrogen therapy in postmenopausal women? A. Ovarian cancer B. Colorectal cancer C. DVT/PE D. Endometrial cancer in those with an intact uterus

Answer - B. Colorectal cancer - Estrogen therapy increases all of the above, but was actually shown to decrease the risk of colorectal cancer.

An elderly male patient presents with complaints of urinary symptoms. He reports that he is having a very difficult time peeing. He states that he has had a history of BPH, but that symptoms have never been this bad. He feels that his bladder is full, but cannot relieve himself. He has a history of hypertension, IBS, and mild dementia. Which of his medications would most likely contribute to thisissue? A. Hydralazine B. Dicyclomine C. Cholestyramine D. Rivastigmine

Answer - B. Dicyclomine - is often used for IBS with cramping and spasms. It is highly anticholinergic and would most likely be exacerbating his BPH by contributing to urinary retention.

Which medication would be an acceptable oral, outpatient option for a patient with suspected MRSA pneumonia? A. Vancomycin B. Doxycycline C. Amoxicillin/clavulanate D. Azithromycin

Answer - B. Doxycycline - can have MRSA activity and would be the best choice in this scenario where an oral option is desired. Vancomycin is typically one of the best choices for MRSA, but can't be given orally.

A 75 year old male presents in a non-shockable asystole. The first round of CPR has been given with the patient remaining in a non-shockable rhythm. Which of the following would be the most appropriate medication to administerfirst? A. Amiodarone 300 mg B. Epinephrine 1 mg C. Lidocaine 250 mg D. Norepinephrine 7.5 mg

Answer - B. Epinephrine 1 mg - Epinephrine is the initial drug of choice in this scenario. Amiodarone or lidocaine may be utilized but not first line. Norepinephrine would not be used in this scenario.

A 58 year old female has a history of breast cancer and is currently being managed on long term tamoxifen. She has been experience significant sadness, lack of ambition, and has recently had thoughts of suicide. The PCP and oncologist would like to avoid anypotential drug interactions that could reduce the effectiveness of the tamoxifen. Which antidepressant would be most appropriate to accomplish this? A. Fluoxetine B. Escitalopram C. Paroxetine D. Bupropion

Answer - B. Escitalopram - would be least likely to impact CYP2D6. Tamoxifen is a prodrug whose active metabolite is created via enzymatic reaction through CYP2D6. Fluoxetine, paroxetine, and bupropion all have the potential to inhibit 2D6.

A 66 year old male presents to the emergency department complaining that he can't goto the bathroom. He hasn't urinated in several hours and feels that he should be going. Which of his medications should bestopped? A. Phenytoin B. Fexofenadine/pseudoephedrine C. Metoprolol D. Extended release oxycodone

Answer - B. Fexofenadine/pseudoephedrine - pseudoephedrine and the alpha agonist effect can contribute acute urinary retention, most often when used in higher dosesand in patients already at risk such as patients with BPH or other urinarycondition.

A patient is taking metoprolol 100 mg BID, furosemide 80 mg BID, spironolactone 50mg once daily, and acetaminophen 1,000 mg four times per day. She is reporting some hearing loss. Which of the following medications is most likely the cause? A. Metoprolol B. Furosemide C. Spironolactone D. Acetaminophen

Answer - B. Furosemide - (loop diuretics) of the above medications would be the most likely to contribute to hearing loss. Aminoglycoside antibiotics, and possibly higher dose aspirin are other common medications that could impact hearing.

NK is a 44 year old male complaining of restless legs. He has tried both ropinirole and pramipexole without success. He also suffers from diabetic gastroparesis and has a lot of nausea. Which of the following would be most appropriate to help with his restlesslegs? A. Quinine B. Gabapentin C. Carbidopa/levodopa D. Acetaminophen

Answer - B. Gabapentin - would be the best alternative to the dopamine agonists in this scenario. Carbidopa/levodopa is well known for causing nausea issues, so given his extensive history, it would probably be best to avoid this medication for RLS. Acetaminophen has a limited to no role in treatment of RLS.

In comparison of medications for hypertensive crisis, which of the following isfalse? A. Avoiding the use of labetalol in patients with asthma isrecommended B. Hydralazine and enalapril are not recommended in pregnantwomen C. Phentolamine could be an option in pheochromocytoma D. Esmolol can be utilized during anesthesia

Answer - B. Hydralazine and enalapril are not recommended in pregnant women - All of the above are true except answer B; hydralazine is a potential option in pregnancy which enalapril is obviously not.

KL is a 22 year old female of presents with excessive menorrhagia. She is wondering what can be done to help with this. Which of the following would be the most appropriate option? A. Addition of Acetaminophen 1,000 mg three times daily duringmenses B. Ibuprofen 800 mg TID during menses C. Oxycodone 5 mg TID prn during menses D. None of the above

Answer - B. Ibuprofen 800 mg TID during menses - NSAIDs are the most appropriate option for menorrhagia. Acetaminophen may help with pain associated with menses, but will not likely help with the excessive blood loss. Oxycodone would be a high risk medication given the age and risk for addition/dependence and would likely have no impact on menorrhagia.

In your work as a researcher, you are tracking the number of new diagnosis of Parkinson's over a 5 year period in a sample of 70 year old patients. Which of the following is this best defined as? A. Null hypothesis B. Incidence C. Prevalence D. Validity

Answer - B. Incidence - The incidence represents the number of new diagnosis over a given amount of time. The prevalence involves the number of patients with that disease at a given point in time. Prevalence gives a sense of how widespread or common a disease is while incidence gives the rate of new occurrences.

A 17 year old female has been initiated on traditional oral contraceptives. She has been experiencing some breakthrough bleeding for the last 6 months. Upon assessment, that bleeding is occurring within the first 1-2 weeks following her period. Which of the following would be most appropriate? A. Increase the progesterone component B. Increase the estrogen component C. Reduce the progesterone component D. Reduce the estrogen component

Answer - B. Increase the estrogen component - if breakthrough bleeding is occurring early to middle timeframe in the cycle would be the most appropriate dosing adjustment to help with this.

A 65 year old has a suspected peptic ulcer. Which of the following is the most common cause of this type of ulcer? A. NSAID use B. Infection with Helicobacter Pylori C. Mechanical injury D. Prednisone use

Answer - B. Infection with Helicobacter Pylori - H. Pylori infection is the most common cause of duodenal ulcers. Medication induced ulcers are the second most common cause (particularly NSAIDs). Mechanical injury and prednisone are possible, but less common occurrences.

Which of the following recommendations would be most appropriate? A. Initiate hydrochlorothiazide 25 mg daily B. Initiate metoprolol succinate 50 mg daily C. Increase Lisinopril to 80 mg daily D. Initiate amlodipine 5 mg daily

Answer - B. Initiate metoprolol succinate 50 mg daily - With the compelling indication of CHF, initiating metoprolol succinate would be the most appropriate recommendation. Lisinopril 40 mg is at the typical max dose. Hydrochlorothiazide doesn't have a CHF compelling indication and the patient doesn't have any edema as well. Amlodipine could exacerbate CHF and doesn't have the compelling indication as well.

A 45 year old presents with permanent atrial fibrillation. Past medical history includes diabetes, hypertension, GERD, and osteoarthritis. He hasn't had any significant cardiac or embolic events. He is initiated on diltiazem for rate control. Which recommendation is most appropriate regarding anticoagulation or antiplatelettherapy? A. Initiate aspirin 81 mg daily B. Initiate rivaroxaban C. Initiate aspirin and clopidogrel D. Initiate extended release aspirin/dipyridamole

Answer - B. Initiate rivaroxaban. - This patient has diabetes and hypertension with atrial fibrillation. This would give this patient a CHADS2Vasc score of 2 which puts them in the moderate to high risk scoring and anticoagulation (specifically DOACs per atrial fibrillations guidelines) would be recommended.

As part of your patient's tuberculosis therapy, he is on pyrazinamide. Which ofthe following monitoring parameters would be most critical? A. Renal function B. LFT's C. Weekly weights D. Fasting blood sugars

Answer - B. LFT's - Pyrazinamide is associated with potential for liver toxicity and LFT's should be monitored closely with ongoing therapy.

Which antihypertensive would be preferred in the setting of erectiledysfunction? A. Hydrochlorothiazide B. Lisinopril C. Metoprolol D. All of the above would not be acceptable in erectiledysfunction

Answer - B. Lisinopril - ACE inhibitors and calcium channel blockers are preferred over diuretics and beta blockers in patients who have erectile dysfunction.

One of your patients has recently been diagnosed with diabetes. Which of the following routine screenings would not be recommended by the American Diabetes Association? A. Foot exams B. Liver monitoring C. Kidney monitoring D. Eye exams

Answer - B. Liver monitoring - is not a recommended routine screening for patients with diabetes. Foot exams, kidney function, and eye exams all are routine screenings that are recommended by the ADA.

Which of the following would be considered the strongest risk factor in determining appropriateness for stress ulcer prophylaxis? A. Use of IV fentanyl for >24 hours B. Mechanical ventilation for >48 hours C. Elevated CPK D. Obesity

Answer - B. Mechanical ventilation for >48 hours - Extended mechanical ventilation has been well proven as a risk factor for stress ulcers. Prophylaxis in this situation would be appropriate.

An 8 year old male is taking methylphenidate 10 mg twice daily for ADHD. From school reports and the dad, the methylphenidate seems to be working through the morning pretty well, but his teacher reports lots of challenges in the afternoon. The dad is concerned that his son is not sleeping at all at night. Upon investigation, the second dose of methylphenidate is being given at 4 PM. Which recommendation would be most appropriate? A. Discontinue the 4 PM dose B. Move the second dose to noon instead of 4 PM C. Start clonidine 0.1 mg at bedtime D. Switch to amphetamine salts to 5 mg twice daily and monitorinsomnia

Answer - B. Move the second dose to noon instead of 4 PM - Moving the second dose to noon makes the most sense from a pharmacokinetic perspective. Switching agents doesn't make a lot of sense since the methylphenidate seems to be working well. If moving the second dose to noon doesn't work with the boy's schedule, it would be appropriate to consider a long acting formulation. Adding another agent (clonidine at bedtime) doesn't seem appropriate when moving the timing of the doses could potentially work.

Regarding vasodilator therapy in acute decompensated heart failure, which ofthe following would be false? A. Nitroprusside dosing is primarily limited by cyanidetoxicity B. Nitroglycerin has a much more vasodilatory effect on the arteries versus the veins C. Oral administration may be patient friendly, but is limited by the slow offsetin the event of hypotension D. Nesiritide has a longer half-life compared to nitroprusside

Answer - B. Nitroglycerin has a much more vasodilatory effect on the arteries versus the veins - All of the above are true with the exception of B. Nitroglycerin has a more pronounced effect on the veins compared to the arteries.

A 33 year old female who is currently breast feeding has been diagnosed with strep throat and is prescribed amoxicillin. With regards to the newborn baby which recommendations is most appropriate? A. Breast feeding should be stopped for the course of theantibiotic B. No changes need to be made in the breast feedingschedule C. Once daily dose could be utilized to minimize exposure with the mom pumping/dumping breast milk 4-6 hours after the dose D. Switching to penicillin would provide a greater level of safety for thebaby

Answer - B. No changes need to be made in the breast feeding schedule - Given the relatively small amount of amoxicillin absorbed into breast milk as well as amoxicillin being a drug frequently used in babies, no changes would need to be made in the amoxicillin.

A 51 year old male is hospitalized with substantial confusion and lethargy. His current labs reveal creatinine = 1.3, BUN = 26, Calcium = 8.9, Sodium = 116, K+ = 3.4, ALT = 15, NH3 = 17. Which of his medications would most likely be contributing to the current symptoms and electrolyte abnormality(ies)? A. Losartan B. Oxcarbazepine C. Metoclopramide D. Carbidopa/levodopa

Answer - B. Oxcarbazepine/carbamazepine - are two common examples of medications that can cause SIADH (hyponatremia). This should be the first medication addressed in this situation.

Which of the following vaccines would not be recommended? A. Influenza B. PPSV-23, C. PCV-13 D. Shingrix

Answer - B. PPSV-23. - He should receive influenza on an annual basis and would be a strong candidate for high dose. He has received zoster vaccine and should not need that again. Shingrix would be indicated now, so you could make the argument that administering that vaccine even if patients have received Zostavax in the past. He has received PPSV-23 twice and once after age 65; another dose is not indicated. He should have PCV-13 as a one time dose.

Which of the following agents would have the highest likelihood of causing neuropathy? A. 5-fluourouracil B. Paclitaxel C. Doxorubicin D. Cyclophosphamide

Answer - B. Paclitaxel - Taxanes and platinum compounds will usually have the highest incidence of neuropathy type side effects of the common chemotherapy agents.

HS has been recently diagnosed with Addison's disease. Which of thefollowing medications would be most appropriate for management? A. Spironolactone B. Prednisone C. Clomiphene D. Metformin

Answer - B. Prednisone - Addison's disease is a disorder where there is too little cortisol and the patient will need replacement with corticosteroids making prednisone the best answer. Remember that hyperkalemia is common in Addison's disorder and adding spironolactone could certainly exacerbate this issue.

A young man with a very complex medical history is hospitalized. He is very confused with substantial CNS changes. He is also very nauseated and is vomiting. Assessing his extremities, he is also having some tremors and twitching. Current labs reveal creatinine 1.23, potassium 3.2, sodium 142, pH 7.59, HCO3- 32. Which of the following drugs if taken in excess would likely cause the current problem? A. Methadone B. Prednisone C. Alcohol D. Salicylates

Answer - B. Prednisone - Salicylates and alcohol intoxication would likely cause acidosis state. Given the elevated pH and HCO3 this would be a metabolic alkalosis situation which can be caused by excessive diuretics, antacids, or steroids. Opioids would be more likely to cause a respiratory acidosis due to hypoventilation

The institutional review board plays an important role within a healthcare system. Which task would not require review by the IRB? A. Retrospective chart reviews B. Reporting of a rare adverse drug event C. Performing randomized controlled trials D. Observationalstudy

Answer - B. Reporting of a rare adverse drug event - All of the following would require review by the IRB with the exception of reporting a rare adverse drug event.

Researchers are looking at a specific gene in the ability to identify patients who will experience thrombocytopenia due to heparin. How well the researchers are able to determine who will get thrombocytopenia from heparin would be classified as ? A. Standard deviation B. Sensitivity C. Specificity D. Correlation

Answer - B. Sensitivity - has to do with the ability to recognize a true positive. In this scenario, the true positive would be a patient with a gene that puts them at risk of thrombocytopenia. Specificity would have to do with the ability to detect true negatives.

A 55 year old male has recent complaints of chest pain and breast tenderness. This is a relatively new symptom and he is questioning if it medication related. Which medication would you most suspect as the cause? A. Fluvoxamine B. Spironolactone C. Gabapentin D. Lovastatin

Answer - B. Spironolactone - is commonly associated with gynecomastia and breast tenderness. This would be the best answer. Fluvoxamine, gabapentin, and lovastatin would likely not cause this adverse effect.

A 68 year old female has received numerous cycles of chemotherapy for breast cancer. The patient has had emesis from chemo in the past, but has done fine withoutmedication therapy in the last few cycles. She is also now on a regimen with low emetogenic potential. Her biggest concern is anticipatory nausea/emesis. Nondrug interventions have failed. Which of the following is the best recommendation for anticipatory nausea and vomiting? A. Initiate sertraline 50 mg once daily B. Start alprazolam 0.5 mg at onset of anticipatorysymptoms C. Initiate dexamethasone 5 mg TID on the day of chemo therapy D. Ondansetron 4 mg with chemo infusion

Answer - B. Start alprazolam 0.5 mg at onset of anticipatory symptoms - With non-drug interventions failing, the best recommendation for anticipatory nausea and vomiting is the use of benzodiazepines on an as needed basis as symptoms arise.

Clinical testing is done on a new diabetes medication that targets fasting blood sugars. The investigators have set up the new medication versus placebo in monitoring those blood sugars. Assuming a normal distribution, which of the following tests would be most appropriate for assessing the change in blood sugars between the two differentgroups? A. Chi-squared B. T-test C. Mann-Whitney test D. Pearson correlation

Answer - B. T-test - The T-test is a statistical test designed for continuous data on a normal distribution when comparing two groups. Blood sugars would be continuous data in this scenario.

Which test/curve is considered an estimator used for determining the percentage of patients living after a specific treatment over a given period of time? A. Wilcoxon B. Kaplan Meier C. ANOVA D. Mann-Whitney

Answer - B. The Kaplan Meier - This curve estimates survival rates over time usually in association with some sort of medical treatment.

Underpowered studies will have a higherthan usual ? A. Type 1 error B. Type 2 error C. Bias D. Correlation

Answer - B. Type 2 error - Underpowered studies will tend to have a high incidence of Type 2 error due to the fact that Type 2 error is when a difference between groups actually exists, but you are unable to detect it. A small sample size often leads to underpowered studies. Bias and correlation won't necessarily be higher than usual and type 1 error involves finding a difference when one doesn't actually exists. This wouldn't necessarily relate to power.

A 45 year old female is diagnosed with a UTI. She is placed on sulfamethoxazole/trimethoprim. Current labs/vitals include: Creatinine = 0.9, Na+ = 134, k+ = 5.4, Calcium = 9.8, Albumin = 4.2, bilirubin = 0.01, pH = 7.36, Hg = 13.6, PLT = 175,000, glucose = 121, AST = 21, ALT = 17. Current medications include: lovastatin, valsartan, atenolol, and aspirin. Which potential drug interaction would be most concerning with the addition of thesulfamethoxazole/trimethoprim? A. Lovastatin B. Valsartan C. Atenolol D. Aspirin

Answer - B. Valsartan - Given the slight elevation in potassium already, the most concerning drug interaction in this scenario would be the potential for trimethoprim to contribute to the hyperkalemia.

A 78 year old male presents to the hospital with fever and chills. Upon assessment by the hospitalist, she notes a heart murmur. The provider orders 3 separate blood cultures and is desiring input on empiric antibiotic therapy for possible endocarditis. Which of the following would be most appropriate to initiate? A. Levofloxacin B. Vancomycin C. Ertapenem D. Sulfamethoxazole/trimethoprim

Answer - B. Vancomycin - Coverage of gram positive organisms like Staph, Strep, and Enterococci is of highest importance if the source of potential endocarditis is unknown. With limited information in this scenario, vancomycin would provide the best coverage of those common organisms that can often cause infective endocarditis.

BM had a recent gout flare. She is taking numerous supplements. Which of the following should be discontinued? A. Garlic B. Ginseng C. Cinnamon D. Niacin

Answer - D. Niacin - is the supplement in this situation that could likely contribute to a gout flare by increasing uric acid. It would be the one that should be discontinued.

A 55 year old has a history of colon cancer. Upon assessment, serum calcium is 14.8 mg/dL. Kidney function is stable and patient has no symptoms of heart failure. Which of the following agents would be most appropriate to treat hypercalcemia. A. Torsemide B. Zoledronic acid C. Metolazone D. Somatostatin

Answer - B. Zoledronic acid - Zoledronic acid along with calcitonin and fluid replacement using normal saline would be the most appropriate course of action in this scenario. Loop diuretics are generally not used in the situation where there isn't fluid overload. Metolazone could actually raise calcium and somatostatin would have no role

Researchers are doing some observations of the medication pravastatin to identify if the medication increases blood sugars in patients with diabetes. They are reviewing fasting blood sugars. The mean fasting blood sugar of the patients is 136. The standard deviation of these patients was 13.9. Under the assumption of a normal distribution, what range of blood sugars would contain 95% of the data? A. Too little information B. 122.1-149.9 C. 108.2-163.8 D. 94.3-177.7

Answer - C. 108.2-163.8 - When a normal distribution is assumed, 95% of the data will be contained within two standard deviations from the mean.

In regards to use of sacubitril/valsartan, which of the following istrue? A. It should be administered with food B. It was demonstrated in studies to reduce the risk of CV death and hospitalization in all patients with heart failure stages of NYHA classesI-IV C. A 36 hour washout period is required if the patient is on an ACE orARB D. Unlike ACE inhibitors, kidney function monitoring is not necessarywith sacubitril/valsartan

Answer - C. A 36 hour washout period is required if the patient is on an ACE or ARB - All statements are false besides C which is true. Sacubitril/valsartan can be administered without respect to meals/food. The drug was shown to be beneficial in patients NYHA class II-IV, and kidney function monitoring is required.

A 16 year old female comes to the ED with increasing shortness of breath. She is currently taking mometasone and has an albuterol inhaler to help manage her asthma. Upon further assessment, she reports using her albuterol inhaler several times per day and is using it multiple nights throughout the week. She is being sent home on oral corticosteroids. Which of the following recommendations is most appropriate in regards to chronic management of her asthma. A. Addition oftiotropium B. Switch albuterol to combinationalbuterol/ipratropium C. Addition of a long acting beta agonist D. Addition oftheophylline

Answer - C. Addition of a long acting beta agonist - In our asthma guidelines, of the above choices, the most appropriate next step would be to add a long acting beta agonist (or possibly increase steroid dose which isn't listed as an option). Theophylline and tiotropium are generally more last line options to manage asthma. Switching to combination SABA/short acting anticholinergic would not be the appropriate course of action. Assessment of inhaler technique and compliance to mometasone would also be important steps to take.

Which of the following would not be an appropriate goal in the setting of acute management of TBI? A. Maintenance of systolic blood pressure greater than 90 B. Prevention of DVT C. Aggressive glucocorticoid therapy to reduce potentialinflammation D. Use of antiepileptic therapy to reduce the risk of status epilepticus

Answer - C. Aggressive glucocorticoid therapy to reduce potential inflammation - All of the above are appropriate goals with the exception of aggressive glucocorticoid therapy. There is some evidence that glucocorticoids may actually worsen outcomes.

EC is a 31 year old male with fibromyalgia. He has tried exercise and other lifestyle modifications to help improve symptoms. His primary care provider would like to consider pharmacotherapy to help manage the pain. Other diagnoses include obesity, prediabetes, and GERD. Which recommendation is most acceptable? A. Acetaminophen B. Ibuprofen C. Amitriptyline D. Tramadol

Answer - C. Amitriptyline - The TCA's, SNRI's, and anticonvulsants like gabapentin or pregabalin would be options to help with fibromyalgia pain. Simple analgesics are generally not that effective and medications with opioid activity are generally discouraged.

A 55 year old has a history of proteinuria due to diabetic nephropathy. Blood pressure is 148/86. He has not tolerated ACE inhibitors or ARBs and is currently untreated for hypertension. Which of the following would be most appropriate? A. Hydrochlorothiazide B. Metoprolol C. Amlodipine D. Spironolactone

Answer - C. Amlodipine (CCB) - would be the best choice in the setting of diabetic nephropathy when an ACE/ARB is not acceptable.

Which possible complication is associated with the use of mineral oil in a patientwith CKD, dysphagia, CHF, and Parkinson's? A. Acute renal failure B. Exacerbation of esophageal edema C. Aspiration pneumonitis D. Hyperkalemia

Answer - C. Aspiration pneumonitis - Mineral oil is avoided in patients with swallowing difficulties as well as the elderly in general to avoid the risk of aspiration pneumonia. Mineral oil if used frequently and chronically enough, does also have the potential of reducing absorption and causing deficiency of fat soluble vitamins.

Which of the following would be the most appropriate empiric therapy for community acquired pneumonia that requires hospitalization? A. Piperacillin/tazobactam + vancomycin B. Vancomycin + azithromycin C. Ceftriaxone + azithromycin D. Levofloxacin + azithromycin

Answer - C. Ceftriaxone + azithromycin - Pip/tazo and vancomycin are typically reserved for hospital acquired pneumonia or instances where there is high suspicion of pseudomonas or MRSA respectively. They would not be appropriate for empiric therapy of community acquired pneumonia requiring hospitalization. Levofloxacin is an alternative option as monotherapy, but typically not used in combination with a macrolide. Ceftriaxone/azithromycin would be the most appropriate selection.

A 41 year old female patient presents with complaints of excessive fatigue and weight gain while taking mirtazapine 45 mg once daily for depression. Mood still remains depressed. Which of the following recommendations would be most appropriate? A. Transition to Paroxetine B. Increase mirtazapine dose to 60 mg daily C. Change mirtazapine to fluoxetine D. Add aripiprazole

Answer - C. Change mirtazapine to fluoxetine - Switching to a more activating type SSRI would be the best option. Paroxetine tends to be more sedating and likely to cause weight gain. She hasn't had a great response to the mirtazapine and increasing the dose to 60 mg likely wouldn't significantly matter as mood remains depressed. Adding aripiprazole and leaving the mirtazapine doesn't make much sense since the mirtazapine is causing adverse effects and hasn't been beneficial.

Which of the following complications is not typically associated with multiplesclerosis? A. Depression B. Visual problems C. Chronic diarrhea D. Bladder dysfunction

Answer - C. Chronic diarrhea - MS presents a variety of potential complications. Typically, constipation and bowel hypomotility tend to be more problematic than issues with chronic diarrhea. Visual problems, depression, and bladder dysfunction are also potential complications. As the disease progresses, MS patients will often require medication therapy for bladder issues, constipation and depression.

A 65 year old male has a history of alcoholism and has just been treated with antibioticsfor his 4th case of spontaneous bacterial peritonitis. His primary provider would like to initiate prophylaxis. Which agent would be most appropriate? A. Amoxicillin/clavulanate B. Clindamycin C. Ciprofloxacin D. Azithromycin

Answer - C. Ciprofloxacin - Ciprofloxacin would be the drug of choice in this scenario as gram negative bacteria like E. coli, Klebsiella, etc. are the species that are most likely to cause bacterial peritonitis. Ciprofloxacin or potentially sulfamethoxazole/trimethoprim would be among the best choices for prophylaxis.

Which of the following are potentially not destroyed by traditional alcoholsanitizers? A. MRSA B. Candida C. Clostridium Difficile D. Adenovirus

Answer - C. Clostridium Difficile - can form spores that alcohol based sanitizers may not be able to destroy. Handwashing with soap and water remains the most effective means of removing the spores from hands.

In regards to management of patients with rheumatoid arthritis, which of the following would be false? A. Assessment for osteoporosis is critical and supplementation with calcium and vitamin D may be important B. Screening is important in females as they are more likely to develop RA than males C. DMARD therapy is not recommended in patients who can be well maintained on low dose NSAID therapy D. Tuberculosis screening is recommended prior to initiation of biologicagents

Answer - C. DMARD therapy is not recommended in patients who can be well maintained on low dose NSAID therapy - DMARD therapy is the gold standard treatment and can delay progression of the disease. Starting these medications early is important, even if the pain from a flare is well managed by NSAIDs or low dose steroids. We would like to use the DMARDs to avoid long term NSAID and steroids if possible due to the significant adverse effect profile of each type of agent.

About 6 months ago, a patient was instructed to take a ½ tablet of her 200 mcg once daily dose of levothyroxine. She was unaware of this change and has been taking the whole200 mcg tablet. Which of the following would not be a likely adverse effect/risk with over supplementation of levothyroxine? A. Osteoporosis B. Tachycardia C. Elevated cholesterol D. Agitation

Answer - C. Elevated cholesterol - Longer term suppressed TSH or higher doses of levothyroxine could lead to osteoporosis, and signs of "hyperthyroid" like tachycardia and agitation. Over supplementation of thyroid replacement would be more likely to reduce cholesterol. Weight loss and elevated temp would also be other possible complications.

A 45 year old male presents with suspected attempted suicide by drug overdose. At this time, it is unclear what drugs the patient overdosed on. The patient does have pinpoint pupils and is unresponsive with shallow respirations. BP is 83/42, pulse is 45, respiratory rate is 8 breaths per minute. Which of the following recommendations is least appropriate? A. Use of isotonic boluses as needed forhypotension B. Empiric administration of IVnaloxone C. Empiric administration offlumazenil D. Get a tox screen as quickly as possible

Answer - C. Empiric Administration of flumazenil - All of the above are appropriate, with the exception of flumazenil. This is generally not recommended as empiric therapy as it can potentially precipitate a seizure. If more information is gathered and benzodiazepine overdose is suspected as well as opioid, then it would be reasonable to consider use of flumazenil.

JS is a 33 year old female who presents with a severe migraine. She states that she experiences 1-2 migraines per week and that they can last up to 24-48 hours sometimes, severely impacting her ability as a parent and employee. She is currently on sumatriptan 100 mg as needed, amlodipine, ethinyl estradiol/norgestimate, and cimetidine for heartburn. She is prescribed Topiramate for migraine prophylaxis. With which of her medications would there be a potential concerning druginteraction? A. Cimetidine B. Amlodipine C. Ethinyl estradiol/norgestimateD. No concerning drug interactions

Answer - C. Ethinyl estradiol/norgestimate - Topiramate does have the potential to interact with birth control and reduce effectiveness. The patient should at a minimum be educated about this and if this is a major concern, alternative contraception and/or an alternative migraine prophylaxis agent should be considered.

With regards to use of aminoglycosides, which of the following isfalse? A. Peak concentrations are monitored to maximize efficacy B. Aminoglycosides should not be combined with beta lactam antibiotics inthe same solution C. For morbidly obese patients, ideal body weight should be used to determine initial weight based dosing D. Lower target concentrations may be desirable if aminoglycosides are being used in combination with other antibiotics for a synergistic effect

Answer - C. For morbidly obese patients, ideal body weight should be used to determine initial weight based dosing - Adjusted body weight equation is typically utilized for aminoglycosides - If the actual weight is greater than 25% of the IBW, the adjusted body weight equation is as follows (ABW = IBW + 0.4(Total body weight - IBW)). Aminoglycosides peaks are monitored for efficacy and toxicity is monitored by trough concentrations. While used together, mixing the two antibiotics (beta-lactam and aminoglycosides) in the same IV bag together can potentially deactivate the drugs.

An obese 29 year old female has been placed on clomiphene to help with infertility. Which of the following would be a common adverse effect to educate herabout? A. Edema B. Myopathy C. Hot flashes D. Skin reactions

Answer - C. Hot flashes - Clomiphene, often used in the treatment of PCOS can cause GI adverse effects and hot flashes as a couple of the common adverse effects.

An elderly female has been placed on desmopressin for nocturia. Which monitoring parameter would be mostimportant? A. Hyperkalemia B. Hypercalcemia C. Hyponatremia D. Hypokalemia

Answer - C. Hyponatremia - Desmopressin has a black box warning for hyponatremia and sodium should be routinely monitored.

With respect to the use of medications in asthma intubation, which of the following isnot true? A. Fentanyl should be avoided in a severely hypotensivepatient B. Lidocaine IV is recommended in patients who have already received albuterol C. IV morphine is the first line opioid of choice in intubation, but fentanyl is an acceptable alternative D. Albuterol is recommended as pretreatment even if the decision has been made to intubate

Answer - C. IV morphine is the first line opioid of choice in intubation, but fentanyl is an acceptable alternative - All of the above are true with the exception of C. Morphine has a slower onset than fentanyl typically by a couple of minutes when given IV. Fentanyl would typically be preferred over the morphine. Lidocaine and albuterol would both be appropriate as above.

SO is a 49 year old female who presents with a rheumatoid arthritis flare. This is her first episode and the PCP has just recently diagnosed her. The PCP has initiated methotrexate 10 mg weekly with folic acid supplementation. SO is still in pain. Current labs reveal hemoglobin = 13.8, creatinine 1.1, GFR >60 mls/min, WBC 5.7, ALT = 17. What would be the most appropriate recommendation for acute pain relief? A. Oxycodone B. Hydroxychloroquine C. Ibuprofen D. Increase methotrexate

Answer - C. Ibuprofen - NSAID for acute flare pain relief would be the most appropriate recommendation. The methotrexate was just started and will take at least a few weeks to begin working. Addition of another DMARD (hydroxychloroquine) does not make sense at this point. Oxycodone is far from a first line agent in the treatment of pain due to RA.

A 66 year old male presents to the ED with a head injury from falling off a ladder. He is diagnosed with intracranial bleeding and there is a desire to do seizure prophylaxis. Which of the following agents would be most appropriate for prophylaxis? A. Lorazepam B. Fosphenytoin C. Levetiracetam D. Ethosuximide

Answer - C. Levetiracetam - for seizure prophylaxis levetiracetam would likely be the best choice in this scenario. Lorazepam and/or Fosphenytoin are typically reserved for active seizures. Ethosuximide would not have a role in seizure prophylaxis in this case.

JS is a 44 year old male with a past medical history of asthma, GERD, HTN, DM, Bipolar disorder and neuropathy. He reports potential signs of lithium toxicity including tremor and nausea and vomiting. Which medication would be most likely to elevate lithium levels? A. Metoprolol B. Duloxetine C. Lisinopril D. Metformin

Answer - C. Lisinopril - ACE inhibitors would most likely cause the elevation in lithium levels. Duloxetine can potentially cause serotonin reactions with lithium, but not as likely to raise levels like the ACE inhibitor. Metformin and metoprolol should not impact lithium levels.

Which of the following statistical tests would be utilized for two unpaired samples containing ordinal data? A. Unpaired t test B. ANOVA C. Mann-Whitney test D. Chi-squared

Answer - C. Mann-Whitney test - Remember that the t test and ANOVA is only for continuous data! Chi-squared is used for nominal data, leaving the Mann-Whitney test for ordinal data as in this situation.

Which of the following is false regarding atrialfibrillation? A. Paroxysmal Afib stops spontaneously without cardioversion and usuallylasts only a few days at most B. When rate control is desired, beta blockers or non-dihydropyridine calcium channel blockers are used preferentially overdigoxin C. Medication therapy with antiarrhythmics is typically first line therapy inthe management of permanent Afib D. Amiodarone should be avoided in thyrotoxicosis that causes atrialfibrillation

Answer - C. Medication therapy with antiarrhythmics is typically first line therapy in the management of permanent Afib - All of the above are true with the exception of C. Rate control with beta-blockers or CCB's are the typical first line therapy. Digoxin may be utilized first line if there is concern for significant orthostasis with the CCB's or beta- blockers.

A 35 year old male has been placed on nortriptyline 25 mg daily to help with depression and insomnia. He and his wife are trying to conceive and he reports recent complaints of sexual dysfunction since starting the nortriptyline. Which of the following would be the most appropriate substitute? A. Sertraline B. Venlafaxine C. Mirtazapine D. Continue nortriptyline for at least 3-6months

Answer - C. Mirtazapine - would have the least potential to cause sexual dysfunction and would likely be the most appropriate transition. Mirtazapine is also sedating particularly at lower doses. SNRI's and SSRI's can also both contribute to sexual dysfunction and it would be best to avoid them at this time. TCA's are also well known to cause sexual dysfunction.

Hydroxyurea has a boxed warning for which of thefollowing? A. Hyperkalemia B. Hypocalcemia C. Myelosuppression D. GI bleeding

Answer - C. Myelosuppression - Hydroxyurea can be used for sickle cell anemia or a few different types of cancer like CML. Myelosuppression is the black box warning for this medication and CBC should be monitored on a routine basis. A 76 year old male has a past medical history of BPH, COPD, CHF, and Barrett's esophagus. His current vaccination record includes: • PPSV-23 6 years ago and 13 years ago • Influenza vaccination 1 year ago • Zoster vaccine 13 years ago

HEDIS measures include numerous measures to help measure the quality or service provided by healthcare institutions. Who is responsible for developing those standards? A. CMS B. FDA C. NCQA D. LEAPFROG

Answer - C. NCQA - National Committee for Quality Assurance is responsible for development and maintenance of HEDIS measures.

In reviewing data, you are recognizing a trend that when blood pressure falls, the risk of cataracts increases. Which of the following best describesthis? A. External validity B. Internal validity C. Negative correlation D. Positive correlation

Answer - C. Negative Correlation - Is described by variables going in opposite directions. As one increases the other decreases. Positive correlation is when one increases, the other increases. External validity has to do with the results of your finding being applicable (or still valid in real life).

Researchers are testing a new antidepressant for improvement in depressive symptoms. They are using the change in PHQ-9 compared to placebo. The PHQ-9 score would be an example of what type of variable? A. Independent B. Nominal C. Ordinal D. Continuous

Answer - C. Ordinal - With PHQ-9, there is a defined order for the scoring (higher indicating a potential greater level of depression), but they are not continuous variables with objective values between each number of the scale.

In regards to healthcare models, capitation payments are best represented by whichof the statements below? A. Payments are tied to the quality of patient outcomes B. Payments are tied to the actual service delivered C. Payments are given based upon the expected cost of a patient witha particular disease D. Each patient is given a set amount of dollars to spend on their healthcare as they see appropriate

Answer - C. Payments are given based upon the expected cost of a patient with a particular disease - Capitation payments involve a set figure of dollars based on a per member per month and estimated average cost incurred for a specific disease. For example, if a patient has diabetes, a payment each month would be made based upon average costs of care for diabetes regardless of how much care is actually provided.

Ambrisentan can be utilized in the management of pulmonary arterial hypertensionbut has a boxed warning/REMS program on risk of? A. Thrombocytopenia B. Hyperphosphatemia C. Risk of serious birth defects D. Hepatic impairment

Answer - C. Risk of serious birth defects - Ambrisentan can cause serious birth defects in pregnant females and does have a REMS program that requires monthly pregnancy tests and appropriate contraception.

In review of the efficacy and safety of a rivastigmine trial, 24% of patients on the higher dose rivastigmine showed improvement via the Alzheimer's disease assessment scale compared to 16% in the placebo group. The P-value was 0.007. Based on the information presented, which of the following istrue? A. Rivastigmine is a safe, beneficial medication for use inAlzheimer's B. Rivastigmine was not shown to be beneficial compared to placebo C. Rivastigmine was shown to be more effective versus placebo D. Rivastigmine by evidence of this information should be the drug of choicefor Alzheimer's dementia

Answer - C. Rivastigmine was shown to be more effective versus placebo - With a p- value less than 0.05, rivastigmine was shown to be more effective than placebo in this scenario. This doesn't indicate that rivastigmine is the drug of choice. The information presented also does not indicate anything about the safety of the medication.

A 54 year old male has had substantial issues with nightmares in association with PTSD. His past medical history includes diabetes, hyperlipidemia, B12 deficiency, obesity, and orthostasis with falls. Current vitals: Temp: 37.1 Celsius, Weight 128 Kg, Ht 5 ft 11inches, Pulse 77 BPM, Blood pressure 99/52; which medication would be most appropriate initial therapy? A. Valproic acid B. Prazosin C. Sertraline D. Olanzapine

Answer - C. Sertraline - Prazosin (an alpha blocker) can be beneficial for nightmares, but with orthostasis, fall risk, and BP already relatively low at 99/52, it should be avoided. SSRI's are indicated in PTSD and first line and would be the best choice in this scenario. Also note the potential for weight gain with olanzapine and valproic acid.

A 62 year old female is reporting an increased in shortness of breath. She is a smoker and has COPD. She has not recently been hospitalized, but physical activity has been limited. Current therapy includes ipratropium/albuterol 2 puffs QID PRN and aclidinium. FEV-1 is 50% of predicted. Which medication change is most appropriate? A. Discontinue aclidinium, startbudesonide B. Add budesonide C. Startformoterol D. Start Roflumilast

Answer - C. Start formoterol - The patient is on an anticholinergic, so the next natural step with the COPD guidelines would be to add a long acting beta agonist. Steroids are usually reserved for frequent exacerbations after LAMA and LABA therapy has been utilized. Roflumilast is typically reserved for refractory cases of COPD.

Which of the following would be the best selection in the treatment of Stenotrophomonas maltophilia? A. Azithromycin B. Ampicillin/sulbactam C. Sulfamethoxazole/trimethoprim D. Ertapenem

Answer - C. Sulfamethoxazole/trimethoprim - Bactrim or levofloxacin as an alternative are the usual drugs of choice to treat Stenotrophomonas maltophilia.

Which of the following characteristics is not associated with chronic kidneydisease? A. Reduced production of endogenous erythropoietin B. Accumulation of serumphosphorus C. Suppressed release of parathyroid hormone D. Hyperkalemia

Answer - C. Suppressed release of parathyroid hormone - All of the above are associated with CKD (usually in the later stages), with the exception of suppressed parathyroid hormone. Hyperparathyroidism is a significant problem in CKD and should be monitored for.

A 65 year old female has been diagnosed with osteoporosis. Which of the following agents should only be used for 2 years? A. Alendronate B. Raloxifene C. Teriparatide D. Denosumab

Answer - C. Teriparatide - is only recommended for two years due to the risk of osteosarcoma which has been proven in rats. Other agents above can be used for longer periods of time. Bisphosphonate therapy may be reassessed at 5 years to see if ongoing use is necessary.

A clinical trial is tasked at trying to determine if PPI's increase osteoporosis risk. Patients are assigned to two different groups. One group is assigned to taking placebo and the other is assigned to taking pantoprazole 40 mg daily. The primary outcome is diagnosis of osteoporosis. Which statistical test would be most appropriate? A. ANOVA B. Mann-Whitney C. Chi-Squared D. T-test

Answer - C. The Chi-Squared test - Is used for nominal data. When separating participants into groups like osteoporosis or non-osteoporosis, this would be nominal data.

Which of the following is false regarding the use of confidence intervals inbiostatistics? A. By convention, a confidence interval is set at the 95% range B. A confidence interval is a range of numbers that the "true" value likely lies between C. The wider the range within a confidence interval, the more precise the study's results are D. A higher proportion of participants in a study typically leads to a narrower confidence interval

Answer - C. The wider the range within a confidence interval, the more precise the study's results are - Is false because the tighter the range of the confidence interval, indicates that the study's results are more precise.

Which of the following would not describe the role of theUSP? A. USP is an independent organization and its recommendations are utilized by the FDA B. USP's goal is related to ensuring the safety and reliability ofmedications produced in the United StatesC. USP acts as enforcement agency if it suspects negligence in the production of pharmaceuticals D. USP has created stands for sterile and non-sterilecompounding

Answer - C. USP can act as enforcement agency if it suspects negligence in the production of pharmaceuticals is incorrect - All of the above are true with the exception of USP having an enforcement role. The FDA uses the standards and they would actually have the ability to reprimand manufacturers or others who aren't are able to follow USP's standards.

Researchers have found that a new medication called Cardios has reduced the risk of heart attack in clinical trials. Patients were separated into two groups and followed for 1 year. 20/1,000 patients experienced a heart attack in the Cardios group while 30/1,000 experienced a heart attack in the placebo group. What is theNNT? A. 30 B. 50 C. 45 D. 100

Answer - D. 100 - NNT is equivalent to 1/Absolute risk reduction. In the placebo group, 3% of the patients had a heart attack while only 2% had a heart attack in the Cardios group. The absolute risk reduction was 1%. 1/0.01 = 100. NNT = 100

A patient presents with the following labs: Na+ = 126 mEq/L, Potassium 3.4 mEq/L, Calcium = 7.6, bicarbonate 17 mEq/L, chloride = 87 mEq/L, creatinine = 1.3, BUN = 33. What is the calculated ion gap? A. 41 B. 117 C. 6 D. 22

Answer - D. 22 - Anion gap = Sodium - (HCO3 + Chloride); In this scenario, that would 126 - (87 + 17). This would represent an elevated anion gap which can help differentiate what is the likely cause of a metabolic acidosis. Usual anion gap is in the 10-12 range.

Use of pantoprazole was studied in patients with Barrett's esophagus over a 5 year period. 200 patients received pantoprazole and 200 patients received placebo. Of those 200 patients in the pantoprazole group, 21 developed esophageal cancer. Of those 200 in the placebo group, 28 developed esophageal cancer. What is the absolute risk reduction of the pantoprazole group? A. 10.5% B. 7% C. 21% D. 3.5%

Answer - D. 3.5% - The absolute risk reduction is going to be 3.5%. If we take placebo groups percentage and subtract the pantoprazole group percentage we will calculate the absolute risk reduction which should be 28/200 - 21/200 = 0.035 or 3.5%. This is a made up example.

Which of the following regarding REMS programs istrue? A. Medications that require a REMS program all have similarrestrictions B. Any medication with potentially serious side effects will have a REMS program C. The REMS program is administered by the CDC D. A REMS program can be initiated at anytime, not just at the time of drug approval

Answer - D. A REMS program can be initiated at anytime, not just at the time of drug approval - A REMS program for a medication can be initiated at any time as new information becomes available. The FDA administers this program and medications often have different requirements/restrictions based upon the risks of the medication. There are lots of medications with serious side effects, but not all medications have a REMS program.

When determining oral bioavailability of a drug, it is best described by comparing what characteristic to the IVformulation? A. Peak concentration B. Time to Cmax C. Elimination rate D. AUC

Answer - D. AUC - In identifying bioavailability of an oral medication, the area under the curve (AUC) for the oral form is divided by the area under the curve for the IV formulation. This does assume that the dose given is the same.

MN is a 52 year old male with hepatic encephalopathy. He has been on lactulose chronically to help with hyperammonemia. He presents with increased confusion and is unable to answer numerous questions on assessment today which is different from his usual. His does report that his rectal area hurts and that he has had some diarrhea. His ammonia level today was 81 compared to 47 just one month ago. He is on lactulose 45mls three times daily. Which of the following would be the most appropriate recommendation? A. Increase lactulose B. Add ciprofloxacin once daily C. Increase protein intake D. Add Rifaximin

Answer - D. Add Rifaximin - Adding Rifaximin makes the most sense in this scenario. The lactulose may even need to be reduced depending upon the severity of the diarrhea, but certainly increasing the dose would not be appropriate at this time. Increasing protein intake would have little effect and possibly increase the risk of hyperammonemia. Ciprofloxacin is not a medication of choice in hepatic encephalopathy and hyperammonemia.

Which of the following does the FDA have regulatory powers over? A. Vaccines B. Medications C. Dietary Supplements D. All of the above

Answer - D. All of the above - The FDA has regulatory powers over all sorts of different products pertaining to food and medical care. Other things included are infant formulas, medical devices and equipment, cosmetics, animal feed, etc.

Which of his medications should bestopped? A. Pantoprazole B. Metoprolol C. Metronidazole D. Celecoxib

Answer - D. Celecoxib - COX-2 inhibitors can contribute/cause acute renal failure and in this scenario, the celecoxib should be discontinued. The other medications could likely be continued assuming resolution and improvement of the kidney failure.

A 34 year old female has had a history of IBS, Barrett's esophagus, insomnia, severe nausea and vomiting, diabetes, and gastroparesis. Her current medications include; sucralfate, omeprazole, ondansetron, chlorpromazine, colestipol, and ramelteon. She presents to the ED with increasing symptoms of achiness, fatigue, fever, and a butterfly shaped rash on her face. Which of the following medications could potentially contribute to thesesymptoms? A. Colestipol B. Ramelteon C. Ondansetron D. Chlorpromazine

Answer - D. Chlorpromazine - Patient is presenting with a possible Lupus type reaction. Chlorpromazine has the potential to cause these types of adverse effects.

Calculated loading dose is dependent upon all of the following factorsexcept: A. Bioavailability B. Volume of Distribution C. Total drug concentration desired D. Clearance

Answer - D. Clearance - In doing a loading dose, the desire is to get the patient to a targeted therapeutic drug concentration quickly. Achieving this initial goal target concentration will not depend upon the clearance. Maintaining this concentration with future doses would depend upon the clearance as well, but not the initial loading dose. Loading Dose = (Volume of distribution X Concentration)/bioavailability

A 52 year old male has developed osteomyelitis. The primary care provider is concerned about coverage of anaerobes. Which of the following agents would have bestcoverage? A. Amoxicillin/clavulanate B. Linezolid C. CefotaximeD. Clindamycin

Answer - D. Clindamycin - If looking for therapy in osteomyelitis and looking for anaerobic coverage, clindamycin would be the best choice given the above options. Metronidazole is another antibiotic that can be beneficial in anaerobic infections, but clindamycin would generally be preferred in osteomyelitis.

A 73 year old male has recently been diagnosed with an ischemic stroke. Upon discharge, the attending physician is asking for an appropriate recommendation for future prevention. Heart sounds/rhythm have been normal and the physician believes the stroke was atherosclerotic in nature. Which of the following would be the most appropriate recommendation? A. Baby aspirin B. Dabigatran C. Warfarin D. Clopidogrel

Answer - D. Clopidogrel - would be the most appropriate given the choice. Extended release aspirin/dipyridamole combination would probably be preferable, but can sometimes not be covered by insurance. Clopidogrel combined with Aspirin has shown an increased risk of bleeding without additional benefit in the setting of atherosclerotic stroke. Anticoagulants are not appropriate in the absence of atrial fibrillation.

A 68 year old female presents to the ED with difficulty breathing. D-Dimer is assessed and substantially elevated. The PCP is suspecting a diagnosis of PE. She has a past history of bipolar disorder, GERD, hot flashes, and hypertension. Current blood pressure is 130/84. Current medications include lamotrigine, asenapine, pantoprazole, conjugated estrogens, and hydrochlorothiazide. Which medication should be discontinuedtoday? A. Lamotrigine B. Asenapine C. Hydrochlorothiazide D. Conjugated estrogens

Answer - D. Conjugated estrogens - With this patient's apparent blood clot (PE), this could have been provoked by the estrogen therapy. This is the primary concern and estrogen should be discontinued.

In assessment of a transplant patient's medications, which of the following shouldbe monitored closely for a potential drug interaction withsimvastatin? A. Prednisone B. Mycophenolate C. Azathioprine D. Cyclosporine

Answer - D. Cyclosporine - and tacrolimus are commonly used for transplantation, but can often have numerous drug interactions through CYP3A4. Mycophenolate, prednisone, and azathioprine should not interact via this mechanism.

A 63 year old female has had 2 vaginal yeast infections in the last 3 months. Which of her medications should be reassessed?A. Fluticasone/salmeterol B. Tiotropium C. Glipizide D. Empagliflozin

Answer - D. Empagliflozin - is a SGLT-2 inhibitor that can increase the risk of UTI's/yeast infections. While inhaled steroids can increase the risk of infection, this would be a localized effect and potentially contribute to thrush.

Which of the following would not be considered PHI? A. Postal address B. Medical record number C. Telephone number D. Gender

Answer - D. Gender - Is too non-specific and would not be considered protected health information. Protected health information is any codes, numbers, words, etc. that could potentially be used to identify a specific individual.

A new patient comes in with pneumonia and a COPD exacerbation. Upon assessment, he is 71 years old and has never been vaccinated. Which statement regarding pneumonia vaccination is correct? A. He should receive both PPSV-23 and PCV-13 today B. He should receive PPSV-23, then PCV-13 1 years afterthat C. He should receive PPSV-23 then PCV-13 1 year later and PPSV-23 5 years later D. He should receive PCV-13, then PPSV-23 1 yearlater

Answer - D. He should receive PCV-13, then PPSV-23 1 year later - 1 dose of PCV13 and PPSV-23 is appropriate for anyone over the age of 65. Two doses of PPSV-23 could be given if given before age 65 and 5 years apart.

Which of the following would not be considered a contraindication in the useof phentermine? A. Hyperthyroidism B. Recent stroke C. Co-administration with tranylcypromine D. History of GI ulcer

Answer - D. History of GI ulcer - MAOI use, stroke, and hyperthyroidism are contraindications for using phentermine in the management of obesity. Some other contraindications include arrhythmias, heart failure, CAD, history of drug abuse, and uncontrolled hypertension. History of GI ulcer is not an absolute contraindication.

Which of the following is not a likely complication with Parkinson's disease? A. Dysphagia B. Depression C. Urinary incontinence D. Hypertension

Answer - D. Hypertension is not common with Parkinson's. - Hypotension is much more likely with Parkinson's disease. Patients should be monitored for low blood pressure and antihypertensives may need to be adjusted over time.

MF is a 61 year old who has been diagnosed with VRE and long term linezolid therapyis warranted. Which of the following monitoring parameters would be least appropriate? Current medications include aspirin, metoprolol, lorazepam as needed, sertraline, pantoprazole, metformin, and sucralfate. A. Serotonin syndrome B. CBC C. Neuropathy D. Hypoglycemia

Answer - D. Hypoglycemia - is not typical with linezolid therapy. Serotonin syndrome, myelosuppression (monitoring CBC), and neuropathy are all potential complications that should be monitored for in this patient. The SSRI should likely be avoided if at all possible.

A 56 year old obese male presents to the emergency department with severe abdominal pain. Upon assessment and review of labwork, the provider feels that he is likely dealing with a case of pancreatitis as amylase is substantially elevated. He asks the pharmacist to review his medications for potential causes. Which of the following would be the most likely culprit to rule out? A. Metformin B. Empagliflozin C. Clonidine D. Liraglutide

Answer - D. Liraglutide - While rare, the GLP-1 analogs are the most likely medication to be tied to the pancreatitis. These medications should be used cautiously and potentially avoided in patients who have had pancreatitis in the past or ruled out in situations where the patient presents with acute pancreatitis.

A nurse has a question regarding QTc prolongation. She is questioning which opioid would be best to use. Which opioid has the highest risk of prolonging the QTinterval? A. Codeine B. Fentanyl C. Oxycodone D. Methadone

Answer - D. Methadone - is one of the worst opioids as far as QTc prolongation and should be avoided in a patient already at risk.

A 1 week old neonate is displaying a high fever and troubling CNS changes. The hospitalist believes that he is dealing with meningitis, likely bacterial. Which of the following bacteria would be least appropriate to target with the selection of empiric antibiotictherapy? A. Listeria monocytogenes B. Escherichia coli C. Strep agalactiae D. Neisseria meningitidis

Answer - D. Neisseria meningitidis - Is the most common cause of meningitis in young adults. This is much less common in neonates. Listeria, E coli, and group B strep are all potential culprits in the case of neonates and should potentially be covered with a 3rd generation cephalosporin and likely add on with ampicillin.

A 76 year old female lives at an assisted living. She has diabetes and her most recent A1C was 7.7. She is on glipizide 5 mg daily. The nurse at the assisted living states that she has had two ER visits in the last 3 months for hypoglycemia. Other diagnosis include; GERD, Hypertension, end-stage CHF, CKD, and constipation. Which recommendation regarding medication management would be most appropriate? A. Add Sitagliptin B. Addition of sliding scale insulin C. Increase glipizide D. No changes to diabetesregimen

Answer - D. No changes to diabetes regimen - With hypoglycemia risk and a diagnosis like end-stage CHF, getting more aggressive with diabetes medications doesn't make much sense. You could make the argument that the sulfonylurea should be discontinued and changed to an alternative like metformin (would need to check on kidney function), or DPP-4. The best answer of the ones presented would be no changes and a less aggressive A1C goal of 8 would be most appropriate.

A 45 year old male has a history of failed suicide attempts including numerous attempts with drug overdose. Which of the following antidepressants would be least appropriateto use in situation? A. Fluvoxamine B. Sertraline C. Venlafaxine D. Nortriptyline

Answer - D. Nortriptyline - TCA's are notorious for causing cardiac changes in overdose and SSRI's and SNRI's are typically considered much safer in higher than therapeutic doses (overdose situation).

Which of the following agencies would have regulatory monitoring over the safety and protection of healthcare workers handling chemotherapy agents? A. FDA B. NIH C. NQF D. OSHA

Answer - D. OSHA - OSHA is the Occupational Safety and Health Administration that is focused on the protection of healthcare workers (and other employees) who may be exposed to hazardous drugs, blood borne pathogens, etc.

A 55 year old male is requesting to use phenylephrine to help with sinus congestion froma recent head cold. He has a history of hypertension and is on amlodipine 5 mg daily, Lisinopril 10 mg daily, and Metoprolol 100 mg BID. Current blood pressure is 139/85. Other past medical history includes history of MI and GI bleed. Which of the following recommendations would be most appropriate? A. Avoid phenylephrine given current blood pressure B. Avoid phenylephrine given history of hypertension C. Avoid phenylephrine given history of MI D. Ok to initiate phenylephrine on a short termbasis

Answer - D. Ok to initiate phenylephrine on a short term basis - Given the decent control of blood pressure, doing phenylephrine short term to try to help with congestion seems reasonable. The MI, history of hypertension would not be contraindications for short term use.

A 78 year old female presents with increasing confusion. There are also complaints of fatigue. Current labs reveal ferritin = 167, creatinine = 1.2, potassium = 4.1, sodium =136, hemoglobin 9.8, HCT = 28.4, B12 = 134, TSH = 0.31, AST = 16, ALT = 18, MCV = 104.9. Which of her medications are most likely contributing to the symptoms? A. Levothyroxine B. Leflunomide C. Naproxen D. Pantoprazole

Answer - D. Pantoprazole - PPI's can impact B12 level and given the low B12 level, confusion, and anemia with elevated MCV, this is the most likely cause. The TSH is out of range a little bit, and should be addressed as well, but likely isn't causing the current symptoms displayed. B12 deficiency can also lead to CNS changes like confusion.

When warfarin is being utilized, which of the following situations will likely requirea higher dose? A. Low albumin levels B. Low intake in green leafy vegetables C. 80 year old patient D. Patient is on carbamazepine

Answer - D. Patient is on Carbamazepine - Carbamazepine is an enzyme inducer and will likely require a higher dose than if a patient wasn't taking this medication. Low albumin, vitamin K intake, and elderly patients will generally impact warfarin by necessitating a lower dose.

With regards to use of oseltamivir in influenza, which of the following isfalse? A. Dose reductions are based upon kidney function B. There have been significant reports of neuropsychiatric events with useof oseltamivir. C. If four individuals in a long term care facility were diagnosed with influenza, prophylaxis for residents within proximity would be warranted D. Patients who have been vaccinated within the last 2 weeks should not receive oseltamivir

Answer - D. Patients who have been vaccinated within the last 2 weeks should not receive oseltamivir - by CDC, 2 or more ill residents in a long term care facility would be classified as an outbreak, dose reductions are based upon kidney function and skin reactions and neuropsychiatric events are warnings/precaution with use. It is not a contraindication to receive oseltamivir near the time of vaccine administration.

A new drug is being tested for use in severe sepsis. In which phase of clinical trials would the highest number of study participants be enrolled to test the drugin? A. Preclinical trials B. Phase 1 C. Phase 2 D. Phase 3

Answer - D. Phase 3 - Clinical trials involves the largest number of patients being tested with the new drug. This is the final step prior to submitting a new drug application (NDA) with the FDA for approval.

HCAHPS surveys would include all of the followingexcept: A. Appropriate discharge information including medicationinformation B. Responsiveness of hospitalstaff C. Willingness to recommend the hospital to someone else D. Possible adverse effects from medication encountered during the hospital stay

Answer - D. Possible adverse effects from medication encountered during the hospital stay - HCAHPS surveys are meant to provide information to the website Hospital Compare for patients and other interested parties to find information on hospitals. Possible adverse effects encountered would not be a survey question contained within HCAHPS

A 54 year old alcoholic patient presents to the ED in severe pain. He has had a long history of gout, hepatic encephalopathy, GI ulcers, and esophageal varices. Current labs reveal creatinine = 1.2, K+ = 5.1, Sodium = 132, Uric acid = 13.6, Hemoglobin = 13.2, platelets = 121. Which of the following would be most appropriate for acute management of the gout flare? A. Allopurinol B. Probenecid C. Indomethacin D. Prednisone

Answer - D. Prednisone - Given the choices here, prednisone would be the best option for acute treatment of the gout flare. Indomethacin would likely be less safe than the prednisone given the significant GI history. The allopurinol and probenecid do have the potential of exacerbating an acute attack. Colchicine would probably be another acceptable option here.

Researchers are studying the mean drop in pulse between metoprolol succinate 50mg daily and metoprolol tartrate 25 mg BID. They are also comparing these two groups to placebo as well. Which of the following would be a dependent variable? A. Metoprolol succinate dose B. Metoprolol tartrate dose C. Placebo control D. Pulse measurements

Answer - D. Pulse Measurements - pulse would be the dependent variable. Independent variables are set by the researchers and in this case, each form of metoprolol and the placebo group are the independent variables.

A 64 year old female is morbidly obese. She is currently on metformin 1,000 mg BID with an A1C of 8.6. The PCP does not want to add another agent that will contribute to weight gain. Which therapy would be most appropriate to try to avoid weight gain? A. Sliding scale to minimize use of scheduledinsulin B. Pioglitazone C. Glimepiride D. Saxagliptin

Answer - D. Saxagliptin - Use of a DPP4 inhibitor would be the best selection to try to avoid weight gain. The glitazones, insulin, and sulfonylureas could all contribute to weight gain.

A 48 year old female has been in a motor vehicle accident with severe trauma. A family member is a healthcare professional and remembers that she has had one otheraccident in the past and has had malignant hyperthermia from a medication, but can't recall the name of it. If neuromuscular blockade is necessary, which of the following agents should likely be avoided or used with extreme caution due to this revelation? A. Neostigmine B. Cisatracurium C. Pancuronium D. Succinylcholine

Answer - D. Succinylcholine - Succinylcholine would be the medication most likely associated with malignant hyperthermia. If utilized in this scenario, the patient should be monitored extremely closely for this or better yet, the medication should be avoided if alternatives exist.

A 65 year old male has had difficulty with urinating. He has had difficulty completely emptying his bladder and it is sometimes painful. He is diagnosed with bacterial prostatitis. What would be the best option? A. Amoxicillin B. Metronidazole C. Azithromycin D. Sulfamethoxazole/trimethoprim

Answer - D. Sulfamethoxazole/trimethoprim - Treatment of bacterial prostatitis involves combating similar bugs to UTI's. Focus should be on gram negatives (i.e. E coli etc.). Cipro, Bactrim, doxycycline are all reasonable options.

A nurse at your institution has been part of a needle stick incident. The needle stick is thought to be from a patient who is HIV positive. Which of the following would be true in regards to PEP therapy? A. The nurse should be given a one time dose of tenofovir, emtricitabine,and dolutegravir B. PEP treatment can be delayed up to 3-5 days if HIV testing is not readily available C. HIV testing for the nurse is not recommended if it is known that the patientis HIV positive D. The nurse should be evaluated for other potential blood borne infectionsand treated as appropriate

Answer - D. The nurse should be evaluated for other potential blood borne infections and treated as appropriate - PEP (Post exposure prophylaxis) therapy, if and when appropriate should be given for 28 days, not simply a one time dose. Initiation of therapy should be as soon as possible, delaying PEP would not be appropriate. HIV testing would be recommended for the nurse.

A 41 year old female has had multiple DVT's including a DVT in a previous pregnancy. She is currently anticoagulated with warfarin. Upon assessment today, it is revealed that she is pregnant. Which of the following would be most appropriate? A. Transition to rivaroxaban B. Transition to baby aspirin C. Continue on warfarin, monitor INR weekly with goal of 2-3 anticipating higher dose of warfarin over time due to increasing volume of distribution D. Transition to enoxaparin

Answer - D. Transition to enoxaparin - Enoxaparin is the anticoagulant of choice in pregnancy. Warfarin is contraindicated and rivaroxaban is fairly new with limited information in pregnancy. Transition to aspirin would not be adequate to cover the patient's anticoagulation needs.

An 88 year old male has increasing complaints of insomnia. His past medical history includes; CHF, COPD, BPH, Constipation, and GERD. Current medications include; digoxin, furosemide, Lisinopril, tiotropium, finasteride, and docusate. Recent vitals; pulse = 62 beats per minute and blood pressure = 132/68. Of the following, which medication would be safest for hisinsomnia? A. Nortriptyline B. Doxepin C. Zolpidem D. Trazodone

Answer - D. Trazodone - Trazodone would be the safest medication for this patient. Drugs with anticholinergic activity (nortriptyline and doxepin) in an 88 year old male with BPH and constipation would not be a good idea. Zolpidem, much like benzodiazepines would not be a first line choice for insomnia if possible due to risk of confusion, falls, and dependence. Trazodone would have a much safer side effect profile.

You are tasked with reviewing patient educational materials for a CHF handout at your institution. Which would improve the readability of yourmaterials? A. Using lengthy sentences to try to explain different aspects of heartfailure B. When explaining data, using exact percentages versusfractions C. Using symbols and abbreviations for larger, more challengingwords D. Utilizing analogies that may relate to the patientpopulation

Answer - D. Utilizing analogies that may relate to the patient population - Analogies that a patient could more easily understand (i.e. the heart is like a pump), would help improve the readability. Symbols/abbreviations tend to confuse patients. Simple fractions like one-half or one-fourth of patients are generally much easily to grasp than exact percentages. Short sentences should be used when possible.

Valproic acid has a boxed warning for which of thefollowing? A. Hepatotoxicity B. Renal failure C. Immunosuppression D. Dermatologic reactions

Answer - Hepatotoxicity - Valproic acid can be used for a few different indications such as seizures, bipolar disorder, and migraines. It has a boxed warning for hepatotoxicity and LFT's should be monitored accordingly.

In which of the following myocardial infarction presentations is the use ofbetablockers potentially not beneficial? A. Patient with a stroke 5 months ago B. MI likely caused by cocaine C. NSTEMI D. Patient with mild COPD on long acting beta agonisttherapy

Answer - MI likely caused by cocaine - LABA therapy is not a contraindication to beta- blocker therapy. Recent stroke should not impact using a beta blocker as well as type of MI (i.e. STEMI vs. NSTEMI). There is some evidence that beta-blockers can worsen outcomes in cocaine induced MI and should likely be avoided. AHA currently recommends avoiding use. A 47 year old patient presents to the emergency department with anuresis and complaining of mid-back and GI pain. Lab Work reveals: • Sodium = 142 • Potassium = 5.3 • Calcium = 9.6 • ALT = 19 • AST = 21 • Creatinine = 3.76 • Hemoglobin 12.9 • WBC = 7.9 Vitals: • Blood pressure 133/72 • Pulse 61 • Temperature = 37.2 Celsius


Ensembles d'études connexes

Micro Econ Ch. 13 Practice Questions

View Set

Screening for Cancer and Other Diseases

View Set

Learnsmarts Questions I got wrong Test 1

View Set